LSAT and Law School Admissions Forum

Get expert LSAT preparation and law school admissions advice from PowerScore Test Preparation.

 BMM2021
  • Posts: 39
  • Joined: Jun 30, 2021
|
#95631
Hi,

I think this question hasn't yet been brought up here because the wrong answers are clearly wrong and the right answer seems to fit well (i.e. it's relatively easy), but despite the fact that I got this one right, I wanted some clarification on the reasoning.

In short, I feel like answer E is more of a Strengthen or Justify answer, not a necessary assumption. Let me know if my reasoning is wrong here.

Although you could argue answer E doesn't make the conclusion 100% true (one could debate what warrants hastiness), I nevertheless feel like the answer is not an implicit assumption within the argument; rather it seems to simply strengthen the conclusion. Given historical evidence that anticapitalist measures have resulted in improved democratization, it would be hasty to criticize similar efforts by governments in certain countries IF those countries were in the midst of transitions away from totalitarianism and towards democracy. The truth of answer E supports the conclusion, but it doesn't seem to me that the author is assuming the truth of answer E. To me, the underlying assumption of the argument was that because anticapitalist developments have had positive effects for democracy in the past, anticapitalist developments do not always deserve criticism, despite their apparent antithetical relationship to democracy. In other words, because action X has not always been a bad thing, it is undue to oppose action X in the present. A principle that states this concept might have instead been the correct "necessary" assumption, in my view.

Anyway, just wondering if my reasoning is off on this?
 Robert Carroll
PowerScore Staff
  • PowerScore Staff
  • Posts: 1787
  • Joined: Dec 06, 2013
|
#95640
BMM2021,

Answer choice (E) is exactly an assumption the argument has to make. The banker has shown that in some circumstances, repressive, anticapitalistic measures are needed. But the only circumstances the banker brought up where such measures are needed are in the transition from a totalitarian regime to a democracy. The conclusion claims that criticism of certain actual anticapitalistic measures being taken currently is hasty. The premises have certainly set up that anticapitalistic measures can be a good idea in some circumstances, but if those circumstances aren't actually happening in the countries that are being criticized, the banker's premises have nothing to do with the actual, present situation of the conclusion.

Basically, the banker is saying "sometimes it's ok to be repressive. So the currently repressive countries aren't that bad." That works only if the "sometimes" situations that justify repression are actually present in this situation, which is what answer choice (E) states.

Robert Carroll
 BMM2021
  • Posts: 39
  • Joined: Jun 30, 2021
|
#95748
Thanks, Robert. Could the relevant part of the argument be summarized like this?

(IF country transitioning from totalitarian to democracy) ---> (THEN temporary anticapitalist measures ok)

If so, would that make the conclusion, in conditional terms, look like this?

(IF Necessary Assumption) ---> (THEN temporary anticapitalist measures ok)

**I'm equating an assessment of hasty judgement by the author to the concept that anticapitalist measures are okay in this conditional.**

Anyway, if that's the case, I understand that the only sufficient element to this equation that we've been given in the stimulus is that transitioning from a totalitarian regime to democracy makes these kind of anticapitalist measures okay, but I don't see why that's a "necessary" sufficient condition? What if another sufficient assumption exists that also makes anticapitalist measures okay? The author is just stating one historically-supported condition in which anticapitalist measures are acceptable, but the author doesn't rule out the possibility of other conditions ("sometimes" situations). I guess I'm just not sure why the historical condition has to be the condition relevant to the "certain governments" in the conclusion. To me, the belief that the sufficient condition mentioned in the stimulus is a necessary assumption for the conclusion is equivalent to confusing the sufficient and necessary terms. We're almost saying because these temporary anticapitalist measures ought to be accepted (or at least not criticized) by the people under "certain governments", it must be the case that these governments are transition from totalitarian regimes to democratic ones:

(anticapitalist measures ok) --> (transitioning from totalitarian to democracy)


Just trying to make sure I fully understand! Thanks
 Adam Tyson
PowerScore Staff
  • PowerScore Staff
  • Posts: 5153
  • Joined: Apr 14, 2011
|
#96558
I think you're there, BMM. The author is saying it's okay to be a little repressive if you are transitioning from totalitarian rule towards democracy, and that therefore the repressive, anti-capitalist measures in certain countries are nothing to be upset about. The author has to assume that those countries are in just such a transition.

Use the negation technique here: what if these certain countries are not in that transition. What if they were just doing what they have always done and were not going through some sort of change for the better? Or what if they had already been democratic and were actually transitioning away from that and towards something more totalitarian? Then the author would look pretty silly here, wouldn't they? That's why this answer is a necessary assumption; without it, their argument lacks any support.

Get the most out of your LSAT Prep Plus subscription.

Analyze and track your performance with our Testing and Analytics Package.